multiplying two normal subgroups is still normal?












3












$begingroup$


Let $G_i triangleleft G_{i+1}$ both subgroups of $G$. Let $N$ be a normal group.



Does $G_iN triangleleft G_{i+1}N$?



Does $(G_iN/N) triangleleft (G_{i+1}N/N)$?



I know that $q:Glongrightarrow G/N$ preserves normality. Hence if $G_iN triangleleft G_{i+1}N$ then their quotients would be normal.



I have tried by considering an element in $gin G_iN$ and $hin G_{i+1}N$ and trying to find out if $ghg^{-1}in G_iN$. But I am a bit confused of those multilpication groups. And that is why I can not play with this expression.










share|cite|improve this question











$endgroup$












  • $begingroup$
    What do you know? What have you tried?
    $endgroup$
    – verret
    Jan 22 at 20:55










  • $begingroup$
    Edited. @verret
    $endgroup$
    – idriskameni
    Jan 22 at 20:58










  • $begingroup$
    Have you heard of the second/third isomorphism theorem?
    $endgroup$
    – DonAntonio
    Jan 22 at 21:00






  • 1




    $begingroup$
    Yes I do. I have been playing with that expression too. $G_i/(G_icap H)triangleleft G_{i+1}/(G_{i+1}cap H)$. But still nothing.
    $endgroup$
    – idriskameni
    Jan 22 at 21:02






  • 1




    $begingroup$
    I have tried a lot of things. Is not that I haven't worked on it. Don't you think so. I mean, it is easy to come and coment these things.
    $endgroup$
    – idriskameni
    Jan 22 at 21:02


















3












$begingroup$


Let $G_i triangleleft G_{i+1}$ both subgroups of $G$. Let $N$ be a normal group.



Does $G_iN triangleleft G_{i+1}N$?



Does $(G_iN/N) triangleleft (G_{i+1}N/N)$?



I know that $q:Glongrightarrow G/N$ preserves normality. Hence if $G_iN triangleleft G_{i+1}N$ then their quotients would be normal.



I have tried by considering an element in $gin G_iN$ and $hin G_{i+1}N$ and trying to find out if $ghg^{-1}in G_iN$. But I am a bit confused of those multilpication groups. And that is why I can not play with this expression.










share|cite|improve this question











$endgroup$












  • $begingroup$
    What do you know? What have you tried?
    $endgroup$
    – verret
    Jan 22 at 20:55










  • $begingroup$
    Edited. @verret
    $endgroup$
    – idriskameni
    Jan 22 at 20:58










  • $begingroup$
    Have you heard of the second/third isomorphism theorem?
    $endgroup$
    – DonAntonio
    Jan 22 at 21:00






  • 1




    $begingroup$
    Yes I do. I have been playing with that expression too. $G_i/(G_icap H)triangleleft G_{i+1}/(G_{i+1}cap H)$. But still nothing.
    $endgroup$
    – idriskameni
    Jan 22 at 21:02






  • 1




    $begingroup$
    I have tried a lot of things. Is not that I haven't worked on it. Don't you think so. I mean, it is easy to come and coment these things.
    $endgroup$
    – idriskameni
    Jan 22 at 21:02
















3












3








3





$begingroup$


Let $G_i triangleleft G_{i+1}$ both subgroups of $G$. Let $N$ be a normal group.



Does $G_iN triangleleft G_{i+1}N$?



Does $(G_iN/N) triangleleft (G_{i+1}N/N)$?



I know that $q:Glongrightarrow G/N$ preserves normality. Hence if $G_iN triangleleft G_{i+1}N$ then their quotients would be normal.



I have tried by considering an element in $gin G_iN$ and $hin G_{i+1}N$ and trying to find out if $ghg^{-1}in G_iN$. But I am a bit confused of those multilpication groups. And that is why I can not play with this expression.










share|cite|improve this question











$endgroup$




Let $G_i triangleleft G_{i+1}$ both subgroups of $G$. Let $N$ be a normal group.



Does $G_iN triangleleft G_{i+1}N$?



Does $(G_iN/N) triangleleft (G_{i+1}N/N)$?



I know that $q:Glongrightarrow G/N$ preserves normality. Hence if $G_iN triangleleft G_{i+1}N$ then their quotients would be normal.



I have tried by considering an element in $gin G_iN$ and $hin G_{i+1}N$ and trying to find out if $ghg^{-1}in G_iN$. But I am a bit confused of those multilpication groups. And that is why I can not play with this expression.







group-theory normal-subgroups






share|cite|improve this question















share|cite|improve this question













share|cite|improve this question




share|cite|improve this question








edited Jan 22 at 20:59









Ethan Bolker

45k553120




45k553120










asked Jan 22 at 20:50









idriskameniidriskameni

759321




759321












  • $begingroup$
    What do you know? What have you tried?
    $endgroup$
    – verret
    Jan 22 at 20:55










  • $begingroup$
    Edited. @verret
    $endgroup$
    – idriskameni
    Jan 22 at 20:58










  • $begingroup$
    Have you heard of the second/third isomorphism theorem?
    $endgroup$
    – DonAntonio
    Jan 22 at 21:00






  • 1




    $begingroup$
    Yes I do. I have been playing with that expression too. $G_i/(G_icap H)triangleleft G_{i+1}/(G_{i+1}cap H)$. But still nothing.
    $endgroup$
    – idriskameni
    Jan 22 at 21:02






  • 1




    $begingroup$
    I have tried a lot of things. Is not that I haven't worked on it. Don't you think so. I mean, it is easy to come and coment these things.
    $endgroup$
    – idriskameni
    Jan 22 at 21:02




















  • $begingroup$
    What do you know? What have you tried?
    $endgroup$
    – verret
    Jan 22 at 20:55










  • $begingroup$
    Edited. @verret
    $endgroup$
    – idriskameni
    Jan 22 at 20:58










  • $begingroup$
    Have you heard of the second/third isomorphism theorem?
    $endgroup$
    – DonAntonio
    Jan 22 at 21:00






  • 1




    $begingroup$
    Yes I do. I have been playing with that expression too. $G_i/(G_icap H)triangleleft G_{i+1}/(G_{i+1}cap H)$. But still nothing.
    $endgroup$
    – idriskameni
    Jan 22 at 21:02






  • 1




    $begingroup$
    I have tried a lot of things. Is not that I haven't worked on it. Don't you think so. I mean, it is easy to come and coment these things.
    $endgroup$
    – idriskameni
    Jan 22 at 21:02


















$begingroup$
What do you know? What have you tried?
$endgroup$
– verret
Jan 22 at 20:55




$begingroup$
What do you know? What have you tried?
$endgroup$
– verret
Jan 22 at 20:55












$begingroup$
Edited. @verret
$endgroup$
– idriskameni
Jan 22 at 20:58




$begingroup$
Edited. @verret
$endgroup$
– idriskameni
Jan 22 at 20:58












$begingroup$
Have you heard of the second/third isomorphism theorem?
$endgroup$
– DonAntonio
Jan 22 at 21:00




$begingroup$
Have you heard of the second/third isomorphism theorem?
$endgroup$
– DonAntonio
Jan 22 at 21:00




1




1




$begingroup$
Yes I do. I have been playing with that expression too. $G_i/(G_icap H)triangleleft G_{i+1}/(G_{i+1}cap H)$. But still nothing.
$endgroup$
– idriskameni
Jan 22 at 21:02




$begingroup$
Yes I do. I have been playing with that expression too. $G_i/(G_icap H)triangleleft G_{i+1}/(G_{i+1}cap H)$. But still nothing.
$endgroup$
– idriskameni
Jan 22 at 21:02




1




1




$begingroup$
I have tried a lot of things. Is not that I haven't worked on it. Don't you think so. I mean, it is easy to come and coment these things.
$endgroup$
– idriskameni
Jan 22 at 21:02






$begingroup$
I have tried a lot of things. Is not that I haven't worked on it. Don't you think so. I mean, it is easy to come and coment these things.
$endgroup$
– idriskameni
Jan 22 at 21:02












2 Answers
2






active

oldest

votes


















2












$begingroup$

I want to change the notation to make stuff clearer: Suppose $Hlhd K< G$, and let $Nlhd G$ (so really $H=G_i$ and $K=G_{i+1}$). Then a proof that $HNlhd KN$ by considering a conjugator is:



Suppose $kin K$, $hin H$, and $n, n'in N$. Firstly, note that
$$n^{-1}k^{-1}(hn')kn=(n^{-1}k^{-1}hkn)cdot (n^{-1}k^{-1}n'kn)$$
Clearly $(n^{-1}k^{-1}n'kn)in N$, while $(n^{-1}k^{-1}hkn)in HN$ by the following:
$$begin{align*}
k^{-1}hk&in H\
Rightarrow k^{-1}hkcdot (k^{-1}hk)^{-1}n^{-1}(k^{-1}hk)&in HN\
n^{-1}(k^{-1}hk)&in HN\
n^{-1}(k^{-1}hk)n&in HN
end{align*}
$$

The result follows.






share|cite|improve this answer











$endgroup$





















    -1












    $begingroup$

    Hint:



    The first one should be almost immediate, and for the second one:



    $$G_iN/Ncong G_i/(G_icap N);,;;; G_{i+1}/(G_{i+1}cap N)cong G_{i+1}N/N$$






    share|cite|improve this answer









    $endgroup$









    • 2




      $begingroup$
      I am afraid that I cannot see how this helps solve the problem. The second one follows directly from the first which, as you say. is straightforward.
      $endgroup$
      – Derek Holt
      Jan 23 at 8:47












    • $begingroup$
      @DerekHolt That isomorphism helps, imo. to see that (1) those quotient groups are well-defined and (2) one is normal in the other. I also think both are more or less straightforward (though this usually depends on the beholder...). Did I miss anything?
      $endgroup$
      – DonAntonio
      Jan 23 at 8:55










    • $begingroup$
      I don't really see how the isomorphisms help prove that one is normal in the other. (The downvote is not from me.)
      $endgroup$
      – Derek Holt
      Jan 23 at 9:11











    Your Answer





    StackExchange.ifUsing("editor", function () {
    return StackExchange.using("mathjaxEditing", function () {
    StackExchange.MarkdownEditor.creationCallbacks.add(function (editor, postfix) {
    StackExchange.mathjaxEditing.prepareWmdForMathJax(editor, postfix, [["$", "$"], ["\\(","\\)"]]);
    });
    });
    }, "mathjax-editing");

    StackExchange.ready(function() {
    var channelOptions = {
    tags: "".split(" "),
    id: "69"
    };
    initTagRenderer("".split(" "), "".split(" "), channelOptions);

    StackExchange.using("externalEditor", function() {
    // Have to fire editor after snippets, if snippets enabled
    if (StackExchange.settings.snippets.snippetsEnabled) {
    StackExchange.using("snippets", function() {
    createEditor();
    });
    }
    else {
    createEditor();
    }
    });

    function createEditor() {
    StackExchange.prepareEditor({
    heartbeatType: 'answer',
    autoActivateHeartbeat: false,
    convertImagesToLinks: true,
    noModals: true,
    showLowRepImageUploadWarning: true,
    reputationToPostImages: 10,
    bindNavPrevention: true,
    postfix: "",
    imageUploader: {
    brandingHtml: "Powered by u003ca class="icon-imgur-white" href="https://imgur.com/"u003eu003c/au003e",
    contentPolicyHtml: "User contributions licensed under u003ca href="https://creativecommons.org/licenses/by-sa/3.0/"u003ecc by-sa 3.0 with attribution requiredu003c/au003e u003ca href="https://stackoverflow.com/legal/content-policy"u003e(content policy)u003c/au003e",
    allowUrls: true
    },
    noCode: true, onDemand: true,
    discardSelector: ".discard-answer"
    ,immediatelyShowMarkdownHelp:true
    });


    }
    });














    draft saved

    draft discarded


















    StackExchange.ready(
    function () {
    StackExchange.openid.initPostLogin('.new-post-login', 'https%3a%2f%2fmath.stackexchange.com%2fquestions%2f3083680%2fmultiplying-two-normal-subgroups-is-still-normal%23new-answer', 'question_page');
    }
    );

    Post as a guest















    Required, but never shown

























    2 Answers
    2






    active

    oldest

    votes








    2 Answers
    2






    active

    oldest

    votes









    active

    oldest

    votes






    active

    oldest

    votes









    2












    $begingroup$

    I want to change the notation to make stuff clearer: Suppose $Hlhd K< G$, and let $Nlhd G$ (so really $H=G_i$ and $K=G_{i+1}$). Then a proof that $HNlhd KN$ by considering a conjugator is:



    Suppose $kin K$, $hin H$, and $n, n'in N$. Firstly, note that
    $$n^{-1}k^{-1}(hn')kn=(n^{-1}k^{-1}hkn)cdot (n^{-1}k^{-1}n'kn)$$
    Clearly $(n^{-1}k^{-1}n'kn)in N$, while $(n^{-1}k^{-1}hkn)in HN$ by the following:
    $$begin{align*}
    k^{-1}hk&in H\
    Rightarrow k^{-1}hkcdot (k^{-1}hk)^{-1}n^{-1}(k^{-1}hk)&in HN\
    n^{-1}(k^{-1}hk)&in HN\
    n^{-1}(k^{-1}hk)n&in HN
    end{align*}
    $$

    The result follows.






    share|cite|improve this answer











    $endgroup$


















      2












      $begingroup$

      I want to change the notation to make stuff clearer: Suppose $Hlhd K< G$, and let $Nlhd G$ (so really $H=G_i$ and $K=G_{i+1}$). Then a proof that $HNlhd KN$ by considering a conjugator is:



      Suppose $kin K$, $hin H$, and $n, n'in N$. Firstly, note that
      $$n^{-1}k^{-1}(hn')kn=(n^{-1}k^{-1}hkn)cdot (n^{-1}k^{-1}n'kn)$$
      Clearly $(n^{-1}k^{-1}n'kn)in N$, while $(n^{-1}k^{-1}hkn)in HN$ by the following:
      $$begin{align*}
      k^{-1}hk&in H\
      Rightarrow k^{-1}hkcdot (k^{-1}hk)^{-1}n^{-1}(k^{-1}hk)&in HN\
      n^{-1}(k^{-1}hk)&in HN\
      n^{-1}(k^{-1}hk)n&in HN
      end{align*}
      $$

      The result follows.






      share|cite|improve this answer











      $endgroup$
















        2












        2








        2





        $begingroup$

        I want to change the notation to make stuff clearer: Suppose $Hlhd K< G$, and let $Nlhd G$ (so really $H=G_i$ and $K=G_{i+1}$). Then a proof that $HNlhd KN$ by considering a conjugator is:



        Suppose $kin K$, $hin H$, and $n, n'in N$. Firstly, note that
        $$n^{-1}k^{-1}(hn')kn=(n^{-1}k^{-1}hkn)cdot (n^{-1}k^{-1}n'kn)$$
        Clearly $(n^{-1}k^{-1}n'kn)in N$, while $(n^{-1}k^{-1}hkn)in HN$ by the following:
        $$begin{align*}
        k^{-1}hk&in H\
        Rightarrow k^{-1}hkcdot (k^{-1}hk)^{-1}n^{-1}(k^{-1}hk)&in HN\
        n^{-1}(k^{-1}hk)&in HN\
        n^{-1}(k^{-1}hk)n&in HN
        end{align*}
        $$

        The result follows.






        share|cite|improve this answer











        $endgroup$



        I want to change the notation to make stuff clearer: Suppose $Hlhd K< G$, and let $Nlhd G$ (so really $H=G_i$ and $K=G_{i+1}$). Then a proof that $HNlhd KN$ by considering a conjugator is:



        Suppose $kin K$, $hin H$, and $n, n'in N$. Firstly, note that
        $$n^{-1}k^{-1}(hn')kn=(n^{-1}k^{-1}hkn)cdot (n^{-1}k^{-1}n'kn)$$
        Clearly $(n^{-1}k^{-1}n'kn)in N$, while $(n^{-1}k^{-1}hkn)in HN$ by the following:
        $$begin{align*}
        k^{-1}hk&in H\
        Rightarrow k^{-1}hkcdot (k^{-1}hk)^{-1}n^{-1}(k^{-1}hk)&in HN\
        n^{-1}(k^{-1}hk)&in HN\
        n^{-1}(k^{-1}hk)n&in HN
        end{align*}
        $$

        The result follows.







        share|cite|improve this answer














        share|cite|improve this answer



        share|cite|improve this answer








        edited Jan 23 at 9:06

























        answered Jan 23 at 8:49









        user1729user1729

        17.4k64193




        17.4k64193























            -1












            $begingroup$

            Hint:



            The first one should be almost immediate, and for the second one:



            $$G_iN/Ncong G_i/(G_icap N);,;;; G_{i+1}/(G_{i+1}cap N)cong G_{i+1}N/N$$






            share|cite|improve this answer









            $endgroup$









            • 2




              $begingroup$
              I am afraid that I cannot see how this helps solve the problem. The second one follows directly from the first which, as you say. is straightforward.
              $endgroup$
              – Derek Holt
              Jan 23 at 8:47












            • $begingroup$
              @DerekHolt That isomorphism helps, imo. to see that (1) those quotient groups are well-defined and (2) one is normal in the other. I also think both are more or less straightforward (though this usually depends on the beholder...). Did I miss anything?
              $endgroup$
              – DonAntonio
              Jan 23 at 8:55










            • $begingroup$
              I don't really see how the isomorphisms help prove that one is normal in the other. (The downvote is not from me.)
              $endgroup$
              – Derek Holt
              Jan 23 at 9:11
















            -1












            $begingroup$

            Hint:



            The first one should be almost immediate, and for the second one:



            $$G_iN/Ncong G_i/(G_icap N);,;;; G_{i+1}/(G_{i+1}cap N)cong G_{i+1}N/N$$






            share|cite|improve this answer









            $endgroup$









            • 2




              $begingroup$
              I am afraid that I cannot see how this helps solve the problem. The second one follows directly from the first which, as you say. is straightforward.
              $endgroup$
              – Derek Holt
              Jan 23 at 8:47












            • $begingroup$
              @DerekHolt That isomorphism helps, imo. to see that (1) those quotient groups are well-defined and (2) one is normal in the other. I also think both are more or less straightforward (though this usually depends on the beholder...). Did I miss anything?
              $endgroup$
              – DonAntonio
              Jan 23 at 8:55










            • $begingroup$
              I don't really see how the isomorphisms help prove that one is normal in the other. (The downvote is not from me.)
              $endgroup$
              – Derek Holt
              Jan 23 at 9:11














            -1












            -1








            -1





            $begingroup$

            Hint:



            The first one should be almost immediate, and for the second one:



            $$G_iN/Ncong G_i/(G_icap N);,;;; G_{i+1}/(G_{i+1}cap N)cong G_{i+1}N/N$$






            share|cite|improve this answer









            $endgroup$



            Hint:



            The first one should be almost immediate, and for the second one:



            $$G_iN/Ncong G_i/(G_icap N);,;;; G_{i+1}/(G_{i+1}cap N)cong G_{i+1}N/N$$







            share|cite|improve this answer












            share|cite|improve this answer



            share|cite|improve this answer










            answered Jan 23 at 8:30









            DonAntonioDonAntonio

            179k1494233




            179k1494233








            • 2




              $begingroup$
              I am afraid that I cannot see how this helps solve the problem. The second one follows directly from the first which, as you say. is straightforward.
              $endgroup$
              – Derek Holt
              Jan 23 at 8:47












            • $begingroup$
              @DerekHolt That isomorphism helps, imo. to see that (1) those quotient groups are well-defined and (2) one is normal in the other. I also think both are more or less straightforward (though this usually depends on the beholder...). Did I miss anything?
              $endgroup$
              – DonAntonio
              Jan 23 at 8:55










            • $begingroup$
              I don't really see how the isomorphisms help prove that one is normal in the other. (The downvote is not from me.)
              $endgroup$
              – Derek Holt
              Jan 23 at 9:11














            • 2




              $begingroup$
              I am afraid that I cannot see how this helps solve the problem. The second one follows directly from the first which, as you say. is straightforward.
              $endgroup$
              – Derek Holt
              Jan 23 at 8:47












            • $begingroup$
              @DerekHolt That isomorphism helps, imo. to see that (1) those quotient groups are well-defined and (2) one is normal in the other. I also think both are more or less straightforward (though this usually depends on the beholder...). Did I miss anything?
              $endgroup$
              – DonAntonio
              Jan 23 at 8:55










            • $begingroup$
              I don't really see how the isomorphisms help prove that one is normal in the other. (The downvote is not from me.)
              $endgroup$
              – Derek Holt
              Jan 23 at 9:11








            2




            2




            $begingroup$
            I am afraid that I cannot see how this helps solve the problem. The second one follows directly from the first which, as you say. is straightforward.
            $endgroup$
            – Derek Holt
            Jan 23 at 8:47






            $begingroup$
            I am afraid that I cannot see how this helps solve the problem. The second one follows directly from the first which, as you say. is straightforward.
            $endgroup$
            – Derek Holt
            Jan 23 at 8:47














            $begingroup$
            @DerekHolt That isomorphism helps, imo. to see that (1) those quotient groups are well-defined and (2) one is normal in the other. I also think both are more or less straightforward (though this usually depends on the beholder...). Did I miss anything?
            $endgroup$
            – DonAntonio
            Jan 23 at 8:55




            $begingroup$
            @DerekHolt That isomorphism helps, imo. to see that (1) those quotient groups are well-defined and (2) one is normal in the other. I also think both are more or less straightforward (though this usually depends on the beholder...). Did I miss anything?
            $endgroup$
            – DonAntonio
            Jan 23 at 8:55












            $begingroup$
            I don't really see how the isomorphisms help prove that one is normal in the other. (The downvote is not from me.)
            $endgroup$
            – Derek Holt
            Jan 23 at 9:11




            $begingroup$
            I don't really see how the isomorphisms help prove that one is normal in the other. (The downvote is not from me.)
            $endgroup$
            – Derek Holt
            Jan 23 at 9:11


















            draft saved

            draft discarded




















































            Thanks for contributing an answer to Mathematics Stack Exchange!


            • Please be sure to answer the question. Provide details and share your research!

            But avoid



            • Asking for help, clarification, or responding to other answers.

            • Making statements based on opinion; back them up with references or personal experience.


            Use MathJax to format equations. MathJax reference.


            To learn more, see our tips on writing great answers.




            draft saved


            draft discarded














            StackExchange.ready(
            function () {
            StackExchange.openid.initPostLogin('.new-post-login', 'https%3a%2f%2fmath.stackexchange.com%2fquestions%2f3083680%2fmultiplying-two-normal-subgroups-is-still-normal%23new-answer', 'question_page');
            }
            );

            Post as a guest















            Required, but never shown





















































            Required, but never shown














            Required, but never shown












            Required, but never shown







            Required, but never shown

































            Required, but never shown














            Required, but never shown












            Required, but never shown







            Required, but never shown







            Popular posts from this blog

            Can a sorcerer learn a 5th-level spell early by creating spell slots using the Font of Magic feature?

            Does disintegrating a polymorphed enemy still kill it after the 2018 errata?

            A Topological Invariant for $pi_3(U(n))$